Diskussion:Potenzmethode

aus Wikipedia, der freien Enzyklopädie
Zur Navigation springen Zur Suche springen

Konvergenzgeschwindigkeit[Quelltext bearbeiten]

Was noch interessant zu erwähnen wäre ist, dass die Konv.geschwindigkeit bei einer symmetrischen Matrix immerhin schon quadratisch ist. (nicht signierter Beitrag von 93.129.40.254 (Diskussion | Beiträge) 21:28, 12. Jul 2009 (CEST))

Hi, ich glaube, das bringst Du mit der Konvergenz der inversen Iteration mit Shift durcheinander. Hier konvergieren sowohl Eigenvektor als auch Eigenwert linear mit dem angegebenen Kontraktionsfaktor.--LutzL 13:30, 13. Jul. 2009 (CEST)[Beantworten]
Hmm, ich denke nicht. Ich referenziere mich einfach mal auf : "Numerik für Ingenieure und Naturwissenschaftler" von W. Dahmen, A.Reusken, Auflage aus dem Jahr 2006, Kapitel 7 "Berechnung von Eigenwerten" -> Unterkapitel 7.5 "Vektoriteration" -> Seite 239. Ich könnte das inklusive Beweis auch mal editieren, schaffe das aber wohl zeitlich vorerst nicht. (nicht signierter Beitrag von 77.180.162.193 (Diskussion | Beiträge) 16:08, 15. Jul 2009 (CEST))

Ich glaube, die Bemerkung über zusätzliche Eigenwerte stimmt nicht. Durch Verwenden von bekommt man keinen anderen Eigenwert, sondern nur den gleichen um verschobenen, richtig?

-- Gerlach 19:54, 8. Jun. 2011 (CEST)[Beantworten]

Das Verfahren für die Matrix beginnt laut Beschreibung mit einem Vektor mit .
Daher sind, wenn man als Eigenwert und Eigenvektor von ermittelt hat, und seine Vielfachen als Startwerte nicht zugelassen bei der Ermittlung weiterer Eigenwert- Eigenvektorpaare von anhand von , denn für die gälte ja . Eine ganze Dimension wird also ausgeblendet. Aber davon mal ab:
Ist nun das durch das Verfahren (auf angewendet) ermittelte weitere Paar, so ist , also .
ist also ein weiterer Eigenwert von . Nur bei sind die beiden gleich, aber 0 ist sowieso ein sehr irreführender "Eigenwert" ;) --Daniel5Ko 00:01, 9. Jun. 2011 (CEST)[Beantworten]
Alles klar. Wenn ich es richtig verstanden habe, findet die Methode angewandt auf den kleinsten Eigenwert von , da ja der (betragsmäßig) größte Eigenwert ist und damit der betragsmäßig größte von .

-- Gerlach 20:03, 14. Jun. 2011 (CEST)[Beantworten]

Google-Matrix[Quelltext bearbeiten]

Nachdem es nun einen eigenen Artikel zum Thema gibt, sollte man hier die Ausführungen auf etwa einen Absatz kürzen, um die Inhalte nicht zu sehr zu doppeln. Viele Grüße, --Quartl (Diskussion) 16:19, 31. Aug. 2013 (CEST)[Beantworten]

korrekt - wie in jeder guten Enzyklopädie. Auch viele Grüße, --Holmium (d) 17:12, 31. Aug. 2013 (CEST)[Beantworten]
Habe den großteil in die Google-Matrix ausgelagert weil auch die separation des Eigenwertes dort schon erläutert wurde, hoffe es passt dort einigermaßen rein. Wenn jemand anderer Meinung ist bitte Melden. grüße zurück ;) --NikelsenH (Diskussion) 17:39, 31. Aug. 2013 (CEST)[Beantworten]
Super, vielen Dank, --Quartl (Diskussion) 18:50, 31. Aug. 2013 (CEST)[Beantworten]

Mehrfache Eigenwerte[Quelltext bearbeiten]

Bei der Stelle "... sofern dieser Eigenwert halbeinfach ist , d.h. es existiert ein index r so dass für die Eigenwerte ..." stimmt wohl das "d.h." nicht ganz: Es könnte doch sein, dass ein r-facher Eigenwert ist, aber hat den gleichen Betrag, also z.B. eine Matrix mit den Eigenwerten 1,1,-1. Oder? -- HilberTraum (Diskussion) 17:32, 31. Aug. 2013 (CEST)[Beantworten]

das d.h. stimmt meiner Meinung nach soweit, es wird ja gefordert , bei 1,1,-1 haut das also nicht hin. Die beschreibung mit worten ist etwas schwammig, muss echt betragsgrößter Eigenwert sein und dazu noch halbbeinfach. Mal schaun ob sich das auch irgendwie in einen deutschen Satz gießen lässt. hoffe es hat sich geklärt LG --NikelsenH (Diskussion) 17:48, 31. Aug. 2013 (CEST)[Beantworten]
Das "d.h." klingt aber so als würde das, was danach kommt, aus "halbeinfach" folgen. Meiner Meinung nach muss man beides zusammen fordern, also halbeinfach und zusätzlich , eben um den Fall 1,1,-1 auszuschließen. -- HilberTraum (Diskussion) 17:55, 31. Aug. 2013 (CEST)[Beantworten]
Danke fürs Überarbeiten, jetzt ists wirklich eindeutig. LG --NikelsenH (Diskussion) 10:18, 1. Sep. 2013 (CEST)[Beantworten]

Der Artikel könnte folgende Fragen klaren:

1) warum konvergiert das Ergebnis im Beispiel von B zunächst gar nicht? Ist das nicht ein Widerspruch zur Konvergenzrate?
2) im Beweis: Was sind die Variablen mit dem Index r? Was meint r? Ist es ein Restwetter senkrecht zum Spektrum?

(nicht signierter Beitrag von 92.73.25.153 (Diskussion) 5. November 2013, 16:47 Uhr)

zu 1) Durch die nichthalbeinfache Vielfachheit von 0,8 ergibt sich in einer Richtung ein Wachstum von , welches erst bei etwa n=10 aufhört.
zu 2) ist beseitigt, der Index r ist jetzt d, die Vielfachheit des halbeinfachen maximalen Eigenwertes.
--LutzL (Diskussion) 14:39, 23. Mai 2014 (CEST)[Beantworten]

Die Folge der Vektoren konvergiert nicht gegen einen Eigenvektor: Sieht man leicht an der komplex diagonalisierbaren Matrix mit Man kann zeigen: jeder Haeufungspunkt der Folge ist ein Eigenvektor zum betragsgroessten Eigenwert, bzw, der Winkel zwischen und dem Eigenraum geht gegen Null. (nicht signierter Beitrag von 132.187.207.64 (Diskussion) 11:23, 23. Mai 2014 (CEST))[Beantworten]

Deshalb steht im Artikel an dieser Stelle die Aussage, dass es nur einen betragsgrößten Eigenwert geben darf. In Deinem Falle haben i und -i beide den Betrag 1, somit fällt es aus der (einfachen) Konvergenzaussage heraus.--LutzL (Diskussion) 14:23, 23. Mai 2014 (CEST)[Beantworten]
Was ist mit dem Beispiel mit Hier hat man doch tatsächlich keine echte Konvergenz, oder? Leider wird der Beweis am Ende, wo die Normierung in Spiel kommt, etwas knapp. -- HilberTraum (Diskussion) 07:17, 24. Mai 2014 (CEST)[Beantworten]
Da fehlt in der Tat etwas, auch zur Bestimmung komplexer Eigenwertpaare aus den letzten zwei oder drei iterierten Vektoren. Man muss nicht nur die Norm des Vektors zur Konvergenz zwingen, sondern auch wenigstens ein Vorzeichen. Eine Variante ist, nicht eine Norm sondern eine Linearform zur Bestimmung von Eigenwert- und -vektorapproximation zu verwenden.--LutzL (Diskussion) 08:51, 24. Mai 2014 (CEST)[Beantworten]